2
0
Fork 0
mirror of https://github.com/MartinThoma/LaTeX-examples.git synced 2025-04-19 11:38:05 +02:00

ÜB 4, Aufgabe 1 mit Lösung hinzugefügt

This commit is contained in:
Martin Thoma 2013-12-02 22:50:18 +01:00
parent fb0c7e39cb
commit 25fc7443b1
5 changed files with 79 additions and 8 deletions

Binary file not shown.

View file

@ -836,7 +836,7 @@ $\qed$
gibt mit $\gamma(0)=x$ und $\gamma(1)=y$.
\end{definition}
\begin{korollar}
\begin{korollar}\label{kor:wegzusammehang-impliziert-zusammenhang}
Sei $X$ ein topologischer Raum.
\begin{enumerate}[label=(\roman*)]

View file

@ -5,3 +5,15 @@
\begin{aufgabe}\label{ub3:aufg1}
\todo{Todo}
\end{aufgabe}
\begin{aufgabe}[Zusammenhang]\label{ub4:aufg1}
\begin{enumerate}[label=(\alph*)]
\item Beweisen Sie, dass eine topologische Mannigfaltigkeit
genau dann wegzusammenhängend ist, wenn sie zusammenhängend
ist
\item Betrachten Sie nun wie in Beispiel~\ref{bsp:mannigfaltigkeit8}
den Raum $X:= (\mdr \setminus \Set{0}) \cup \Set{0_1, 0_2}$
versehen mit der dort definierten Topologie. Ist $X$
wegzusammenhängend?
\end{enumerate}
\end{aufgabe}

View file

@ -87,20 +87,20 @@ U_i = \Set{(x_0: \dots : x_n) \in \praum^n(\mdr) | x_i \neq 0} &\rightarrow \mdr
Zusammenhangskomponenten, wenn man einen Punkt entfernt.
\item $V_2 = \Set{(x,y) \in \mdr^2 | x^3 = y^2}$ ist eine
Mannigfaltigkeit.
\item $X = (\mdr \setminus \Set{0}) \cup (O_1, O_2)$
\item $X = (\mdr \setminus \Set{0}) \cup (0_1, 0_2)$ \label{bsp:mannigfaltigkeit8}
\[U \subseteq X \text{ offen } \gdw
\begin{cases}
U \text{ offen in } \mdr \setminus \Set{0}, &\text{falls } O_1 \notin U, O_2 \in U\\
\exists \varepsilon > 0 \text{ mit } (-\varepsilon, \varepsilon) \subseteq U &\text{falls } O_1 \in U, O_2 \in U
U \text{ offen in } \mdr \setminus \Set{0}, &\text{falls } 0_1 \notin U, 0_2 \in U\\
\exists \varepsilon > 0 \text{ mit } (-\varepsilon, \varepsilon) \subseteq U &\text{falls } 0_1 \in U, 0_2 \in U
\end{cases}\]
Insbesondere sind $(\mdr \setminus \Set{0}) \cup \Set{O_1}$
und $(\mdr \setminus \Set{0}) \cup \Set{O_2}$ offen und
Insbesondere sind $(\mdr \setminus \Set{0}) \cup \Set{0_1}$
und $(\mdr \setminus \Set{0}) \cup \Set{0_2}$ offen und
homöomorph zu $\mdr$.
\underline{Aber:} $X$ ist nicht hausdorffsch!
Denn es gibt keine disjunkten Umgebungen von $O_1$ und
$O_2$.
Denn es gibt keine disjunkten Umgebungen von $0_1$ und
$0_2$.
\item $\GL_n(\mdr)$ ist eine Mannigfaltigkeit der Dimension
$n^2$, weil offene Teilmengen von $\mdr^{n^2}$ eine
Mannigfaltigkeit bilden.

View file

@ -95,3 +95,62 @@
\end{beweis}
\end{enumerate}
\end{solution}
\begin{solution}[\ref{ub4:aufg1}]
\begin{enumerate}[label=(\alph*)]
\item \textbf{Vor.:} Sei $M$ eine topologische Mannigfaltigkeit.\\
\textbf{Beh.:} $M$ ist wegzusammehängend $\gdw M$ ist zusammenhängend
\begin{beweis}
\enquote{$\Rightarrow$}: Da $M$ insbesondere ein
topologischer Raum ist folgt diese Richtung direkt
aus Korollar~\ref{kor:wegzusammehang-impliziert-zusammenhang}.
\enquote{$\Leftarrow$}: Seien $x,y \in M$ und
\[Z := \Set{z \in M | \exists \text{Weg von } x \text{ nach } z}\]
Es gilt:
\begin{enumerate}[label=(\roman*)]
\item $Z \neq \emptyset$, da $M$ lokal wegzusammenhängend ist
\item $Z$ ist offen, da $M$ lokal wegzusammenhängend ist
\item $Z^C := \Set{\tilde{z} \in M | \nexists \text{Weg von } x \text{ nach } \tilde{z}}$ ist offen
Da $M$ eine Mannigfaltigkeit ist, existiert zu jedem
$\tilde{z} \in Z^C$ eine offene und wegzusammenhängende Umgebung
$U_{\tilde{z}} \subseteq M$.
Es gilt sogar $U_{\tilde{z}} \subseteq Z^C$, denn
gäbe es ein $U_{\tilde{z}} \ni \overline{z} \in Z$,
so gäbe es Wege $\gamma_2:[0,1] \rightarrow M, \gamma_2(0) = \overline{z}, \gamma_2(1) = x$
und $\gamma_1:[0,1] \rightarrow M, \gamma_1(0) = \tilde{z}, \gamma_1(1) = \overline{z}$.
Dann wäre aber
\[\gamma:[0,1] \rightarrow M,\;\;\; \gamma(x) = \begin{cases}
\gamma_1(2x) &\text{falls } 0 \leq x \leq \frac{1}{2}\\
\gamma_2(2x-1) &\text{falls } \frac{1}{2} < x \leq 1
\end{cases}\]
ein stetiger Weg von $\tilde{z}$ nach $x$
$\Rightarrow$ Widerspruch.
Da $M$ zusammenhängend ist und $M = \underbrace{Z}_{\mathclap{\text{offen}}} \cup \underbrace{Z^C}_{\mathclap{\text{offen}}}$,
sowie $Z \neq \emptyset$ folgt $Z^C = \emptyset$.
Also ist $M=Z$ wegzusammenhängend.$\qed$
\end{enumerate}
\end{beweis}
\item \textbf{Beh.:} $X$ ist wegzusammenhängend.\\
\begin{beweis}
$X:= (\mdr \setminus \Set{0}) \cup \Set{0_1, 0_2}$
und $(\mdr \setminus \Set{0}) \cup \Set{0_2}$ sind
homöomorph zu $\mdr$. Also sind die einzigen kritischen
Punkte, die man nicht verbinden können könnte
$0_1$ und $0_2$.
Da $(\mdr \setminus \Set{0}) \cup \Set{0_1}$ homöomorph
zu $\mdr$ ist, exisitert ein Weg $\gamma_1$ von $0_1$
zu einem beliebigen Punkt $a \in \mdr \setminus \Set{0}$.
Da $(\mdr \setminus \Set{0}) \cup \Set{0_2}$ ebenfalls
homöomorph zu $\mdr$ ist, existiert außerdem ein Weg
$\gamma_2$ von $a$ nach $0_2$. Damit existiert ein
(nicht einfacher)
Weg $\gamma$ von $0_1$ nach $0_2$. $\qed$
\end{beweis}
\end{enumerate}
\end{solution}